无穷级数

常数项级数

概念

给定数列

u1,u2,,un,u_1, u_2, \cdots, u_n, \cdots

形如

u1++un+u_1 + \cdots + u_n + \cdots

的和式称为 (常数项)无穷级数,简称 (常数项)级数,记作

n=1un\sum_{n=1}^{\infty} u_n

unu_n 称为级数的通项

作级数前 nn 项和的函数 SnS_n 称为级数的部分和,即

Sn=u1+u2++un=k=1nukS_n = u_1 + u_2 + \cdots + u_n = \sum_{k=1}^{n} u_k

如果级数 n=1un\displaystyle \sum_{n=1}^{\infty}u_n 部分和数列 {Sn}\{S_n\} 有极限 SS,即 limnSn=S\displaystyle \lim_{n \to \infty} S_n = S,则称级数 n=1un\displaystyle \sum_{n=1}^{\infty} u_n 收敛,并称 SS 为级数的

如果数列 {Sn}\{S_n\} 无极限,则称级数 n=1un\displaystyle \sum_{n=1}^{\infty} u_n 发散

当级数 n=1un\displaystyle \sum_{n=1}^{\infty} u_n 收敛时,其部分和与和 SS 的差 rn=SSnr_n = S - S_n 称为级数的余项

rn|r_n| 是用部分和 SnS_n 代替级数的和 SS 所产生的误差。

性质

若级数 n=1un\displaystyle \sum_{n=1}^{\infty} u_n 收敛,kk 为常数,则 n=1kun\displaystyle \sum_{n=1}^{\infty} ku_n 也收敛,且

n=1kun=kn=1un\sum_{n=1}^{\infty} ku_n = k \sum_{n=1}^{\infty} u_n

若级数 n=1un\displaystyle \sum_{n=1}^{\infty} u_nn=1vn\displaystyle \sum_{n=1}^{\infty} v_n 都收敛,则 n=1(un±vn)\displaystyle \sum_{n=1}^{\infty} (u_n \pm v_n) 也收敛,且

n=1(un±vn)=n=1un±n=1vn\sum_{n=1}^{\infty} (u_n \pm v_n) = \sum_{n=1}^{\infty} u_n \pm \sum_{n=1}^{\infty} v_n

若级数 n=1un\displaystyle \sum_{n=1}^{\infty} u_n 收敛,则对这级数的项任意加括号所成的级数

(u1++u1i)++(uin1+1++uin)+(u_1 + \cdots + u_{1_i}) + \cdots + (u_{i_{n-1}+1} + \cdots + u_{i_n}) + \cdots

仍收敛,且和不变。


加括号后收敛,未必收敛。如

(11)+(11)+=01(11)(11)=1\begin{aligned} (1 - 1) + (1 - 1) + \cdots = 0\\ 1 - (1 - 1) - (1 - 1) - \cdots = 1 \end{aligned}

级数收敛的必要条件

级数 n=1un\displaystyle \sum_{n=1}^{\infty} u_n 收敛的必要条件limnun=0\displaystyle \lim_{n \to \infty} u_n = 0

级数中去掉、加上或改变有限项,级数的敛散性不变。

柯西收敛原理

级数 n=1un\displaystyle \sum_{n=1}^{\infty} u_n 收敛的充分必要条件是:对任意 ε>0\varepsilon > 0,存在 NNN \in \N,使得 n>m>Nn > m > N 时,有

um+1++un<ε|u_{m+1} + \cdots + u_n| < \varepsilon

即级数的部分和 SnS_nSmS_m 之差 SnSm<ε|S_n - S_m| < \varepsilon

正项级数

正项级数

如果级数 n=1un\displaystyle \sum_{n=1}^{\infty} u_n 的每一项 un0u_n \ge 0,则称 n=1un\displaystyle \sum_{n=1}^{\infty} u_n正项级数

正项级数 n=1un\displaystyle \sum_{n=1}^{\infty}u_n 收敛的充分必要条件是:它的部分和数列 {Sn}\{S_n\} 有上界。

比较判别法:若两个正项级数 n=1un\displaystyle \sum_{n=1}^{\infty} u_nn=1vn\displaystyle \sum_{n=1}^{\infty} v_n 对任意 nn 都有 unvnu_n \le v_n,则

  1. n=1vn\displaystyle \sum_{n=1}^{\infty} v_n 收敛时,则 n=1un\displaystyle \sum_{n=1}^{\infty} u_n 也收敛;
  2. n=1un\displaystyle \sum_{n=1}^{\infty} u_n 发散时,则 n=1vn\displaystyle \sum_{n=1}^{\infty} v_n 也发散。

若存在正整数 NN 及常数 C>0C > 0 使得

0unCvn,(n>N)0 \le u_n \le C v_n, \quad (n > N)

成立,则

  1. n=1vn\displaystyle \sum_{n=1}^{\infty} v_n 收敛,则 n=1un\displaystyle \sum_{n=1}^{\infty} u_n 也收敛;
  2. n=1un\displaystyle \sum_{n=1}^{\infty} u_n 发散,则 n=1vn\displaystyle \sum_{n=1}^{\infty} v_n 也发散。

p-级数

讨论 pp-级数

n=11np=1+12p+\sum_{n=1}^{\infty} \frac{1}{n^p} = 1 + \frac{1}{2^p} + \cdots

的敛散性。其中 pp 为常数。


p1p \le 1 时,1np1n\dfrac{1}{n^p} \ge \dfrac{1}{n},而调和级数 n=11n\displaystyle \sum_{n=1}^{\infty} \dfrac{1}{n} 发散,比较判别法知 p1p \le 1pp-级数发散。

p>1p > 1 时,有

nn+11xp ⁣dx1npn1n1xp ⁣dx\int_n^{n+1} \dfrac{1}{x^p} \d x \le \dfrac{1}{n^p} \le \int_{n-1}^n \dfrac{1}{x^p} \d x

从而 pp-级数的部分和有

1+2n+11xp ⁣dx1+i=2n1np1+1n1xp ⁣dx=1+11px1p1n=1+1p1(1n1p)<1+1p1\begin{aligned} 1 + \int_2^{n+1} \dfrac{1}{x^p} \d x \le 1 + \sum_{i=2}^{n}\dfrac{1}{n^p} &\le 1 + \int_1^n \dfrac{1}{x^p} \d x\\ &= 1 + \dfrac{1}{1-p} x^{1-p} \as_1^{n}\\ &= 1 + \dfrac{1}{p-1} \left( 1 - n^{1-p} \right)\\ &< 1 + \dfrac{1}{p-1} \end{aligned}

因此 pp-级数部分和有上界,故 pp-级数收敛。

比较判别法的极限形式

设级数 n=1un\displaystyle \sum_{n=1}^{\infty} u_nn=1vn\displaystyle \sum_{n=1}^{\infty} v_n 都是正项级数,且 limnunvn=l\lim\limits_{n \to \infty} \dfrac{u_n}{v_n} = l,则

  1. 0l<+0 \le l < +\infty ,且级数 n=1vn\displaystyle \sum_{n=1}^{\infty} v_n 收敛,则 n=1un\displaystyle \sum_{n=1}^{\infty} u_n 也收敛;
  2. 0<l+0 < l \le +\infty[1],且级数 n=1vn\displaystyle \sum_{n=1}^{\infty} v_n 发散,则 n=1un\displaystyle \sum_{n=1}^{\infty} u_n 也发散。

  1. 不严谨的写法,仅作为简记。 ↩︎

比较判别法的推论

对正项级数 n=1un\displaystyle\sum_{n=1}^{\infty} u_nn=1vn\displaystyle\sum_{n=1}^{\infty} v_n,若有 un+1unvn+1vn\dfrac{u_{n+1}}{u_n} \le \dfrac{v_{n+1}}{v_n},则

  1. n=1vn\displaystyle\sum_{n=1}^{\infty} v_n 收敛,则 n=1un\displaystyle\sum_{n=1}^{\infty} u_n 收敛
  2. n=1un\displaystyle\sum_{n=1}^{\infty} u_n 发散,则 n=1vn\displaystyle\sum_{n=1}^{\infty} v_n 发散
证明

由题意有 un+1vn+1unvn\dfrac{u_{n+1}}{v_{n+1}} \le \dfrac{u_n}{v_n},设

limnunvn=ρ\lim_{n \to \infty} \dfrac{u_{n}}{v_{n}} = \rho

unvnρ\dfrac{u_n}{v_n} \ge \rho,即 unρvnu_n \ge \rho v_n,从而由比较判别法知结论成立。

达朗贝尔判别法(比值判别法)

设级数 n=1un\displaystyle \sum_{n=1}^{\infty} u_n 是正项级数,且 limnun+1un=ρ\lim\limits_{n \to \infty} \dfrac{u_{n+1}}{u_n} = \rho,则

  1. ρ<1\rho < 1,则 n=1un\displaystyle \sum_{n=1}^{\infty} u_n 收敛;
  2. ρ>1\rho > 1,则 n=1un\displaystyle \sum_{n=1}^{\infty} u_n 发散;
  3. ρ=1\rho = 1,则级数可能收敛,可能发散。

达朗贝尔判别法证明

仅证明 ρ<1\rho < 1 时级数收敛。

limnun+1un=ρ<1\displaystyle \lim_{n \to \infty} \dfrac{u_{n+1}}{u_n} = \rho < 1,存在 NNξ\xi,使得任意 n>Nn > N 时有 un+1un<ξ<1\dfrac{u_{n+1}}{u_n} < \xi < 1

un+1<ξun<<ξnNuN+1u_{n+1} < \xi u_n < \cdots < \xi^{n-N} u_{N+1}

ξnNuN+1=(uN+1ξN+1)与 n 无关,为常数ξn+1=Cξn+1\begin{aligned} \xi^{n-N} u_{N+1} &= \overbrace{\left(\dfrac{u_{N+1}}{\xi^{N+1}}\right)}^{\text{与 } n \text{ 无关,为常数}} \xi^{n+1}\\ &= C \xi^{n+1} \end{aligned}

un+1<Cξn+1u_{n+1} < C \xi^{n+1}

进而 n=1un=n=1Nun+n=N+1un\displaystyle \sum_{n=1}^{\infty} u_n = \sum_{n=1}^{N}u_n + \sum_{n=N+1}^{\infty}u_n 收敛。

柯西判别法(根值判别法)

设级数 n=1un\displaystyle \sum_{n=1}^{\infty} u_n 是正项级数,且 limnunn=ρ\lim\limits_{n \to \infty} \sqrt[n]{u_n} = \rho,则

  1. ρ<1\rho < 1,则 n=1un\displaystyle \sum_{n=1}^{\infty} u_n 收敛;
  2. ρ>1\rho > 1,则 n=1un\displaystyle \sum_{n=1}^{\infty} u_n 发散;
  3. ρ=1\rho = 1,则级数可能收敛,可能发散。

柯西判别法证明

仅证明 ρ<1\rho < 1 时级数收敛。

limnunn=ρ<1\displaystyle \lim_{n \to \infty} \sqrt[n]{u_n} = \rho < 1,存在 NNξ\xi,使得任意 n>Nn > N 时有 unn<ξ<1\sqrt[n]{u_n} < \xi < 1

un<ξnu_n < \xi^n

进而 n=1un\displaystyle \sum_{n=1}^{\infty} u_n 收敛。

柯西积分判别法

设级数 n=1un\displaystyle \sum_{n=1}^{\infty} u_n 是正项级数,若存在一个连续的单调减少的正值函数 f(x)f(x),使得

un=f(n),n=1,2,u_n = f(n), \quad n = 1, 2, \cdots

n=1un\displaystyle \sum_{n=1}^{\infty} u_n1+f(x) ⁣dx\displaystyle \int_1^{+\infty} f(x) \d x 具有相同的敛散性。


柯西积分判别法证明

uk1=k1kuk1 ⁣dxk1kf(x) ⁣dxk1kuk ⁣dx=uk\begin{aligned} u_{k-1} &= \int^{k}_{k-1} u_{k-1} \d x\\ &\ge \int^{k}_{k-1} f(x) \d x\\ &\ge \int^{k}_{k-1} u_k \d x\\ &= u_k \end{aligned}

k=2nuk1k=2nk1kf(x) ⁣dx=1nf(x) ⁣dxk=2nuk\begin{aligned} \sum_{k=2}^{n}u_{k-1} &\ge \sum_{k=2}^{n}\int_{k-1}^{k}f(x) \d x = \int_{1}^{n}f(x) \d x\\ &\ge \sum_{k=2}^{n}u_k \end{aligned}

从而 n=1un\displaystyle \sum_{n=1}^{\infty} u_n1+f(x) ⁣dx\displaystyle \int_1^{+\infty} f(x) \d x 具有相同的敛散性。

拉阿贝(拉比,Raabe)判别法

对正项级数 n=1un\displaystyle\sum_{n=1}^{\infty} u_n,且 limnn(unun+11)=p\lim\limits_{n \to \infty} n\left(\dfrac{u_n}{u_{n+1}} - 1\right) = p,则

  1. p>1p > 1,则 n=1un\displaystyle\sum_{n=1}^{\infty} u_n 收敛;
  2. p<1p < 1,则 n=1un\displaystyle\sum_{n=1}^{\infty} u_n 发散。
  3. p=1p = 1,则级数可能收敛,可能发散。
证明

仅证明 p>1p > 1 时级数收敛。则存在 1<r<p1 < r < p,使得

limnn((1+1n)r1)=limt0+(1+t)r1t=limt0+r(1+t)r11=r\begin{aligned} \lim_{n\to \infty }n \left( \left(1 + \dfrac{1}{n}\right)^r - 1 \right) &= \lim\limits_{t \to 0^{+}} \dfrac{(1 + t)^r - 1}{t}\\ &= \lim\limits_{t \to 0^{+}} \dfrac{r(1 + t)^{r-1}}{1}\\ &= r \end{aligned}

从而有

limnn(unun+11)=p>r=limnn((1+1n)r1)\lim\limits_{n \to \infty} n\left(\dfrac{u_n}{u_{n+1}} - 1\right) = p > r = \lim\limits_{n\to \infty }n \left( \left(1 + \dfrac{1}{n}\right)^r - 1 \right)

因此当 nn 充分大时,有 unun+1>(n+1)rnr\dfrac{u_n}{u_{n+1}} > \dfrac{(n+1)^{r}}{n^r},即 un+1un<1(n+1)r1nr\dfrac{u_{n+1}}{u_n} < \dfrac{\frac{1}{(n+1)^r}}{\frac{1}{n^r}},因为 r>1r > 1,比较判别法知 n=1un\displaystyle\sum_{n=1}^{\infty} u_n 收敛。

判断正项级数

n=1n!ennn+p\sum_{n=1}^{\infty} \dfrac{n! \e^n}{n^{n+p}}

的敛散性。


解答

anan+1=n!en+1nn+p(n+1)n+p+1(n+1)!en+1=(1+1n)n+pe=exp((n+p)ln(1+1n)1)\begin{aligned} \dfrac{a_n}{a_{n+1}} &= \dfrac{n! \e^{n+1}}{n^{n+p}}\cdot \dfrac{(n+1)^{n+p+1}}{(n+1)! \e^{n+1}}\\ &= \dfrac{\left(1+\frac{1}{n}\right)^{n+p}}{\e}\\ &= \exp \left( (n+p)\ln \left(1 + \dfrac{1}{n}\right) - 1 \right) \end{aligned}

n(anan+11)=n[exp((n+p)ln(1+1n)1)1]n[exp((n+p)(1n12n2+13n3+O(1n3))1)1]n[exp(p12n+13p2n2+O(1n2))1]p12+13p2n+O(1n)p12\begin{aligned} n\left( \dfrac{a_n}{a_{n+1}} - 1 \right) &= n\left[ \exp \left( (n+p)\ln \left(1 + \dfrac{1}{n}\right) - 1 \right) - 1 \right]\\ &\to n\left[ \exp \left((n + p)\left( \dfrac{1}{n} - \dfrac{1}{2n^2} + \dfrac{1}{3n^3} + O\left(\dfrac{1}{n^3}\right)\right) - 1 \right) - 1 \right]\\ &\to n\left[ \exp \left(\dfrac{p - \frac{1}{2}}{n} + \dfrac{\frac{1}{3}-\frac{p}{2}}{n^2} + O\left(\dfrac{1}{n^2}\right)\right) - 1 \right]\\ &\to p - \dfrac{1}{2} + \dfrac{\frac{1}{3}-\frac{p}{2}}{n} + O\left(\dfrac{1}{n}\right)\\ &\to p - \dfrac{1}{2} \end{aligned}

拉阿贝判别法知 p>32p > \dfrac{3}{2} 时收敛,p<32p < \dfrac{3}{2} 时发散。


斯特林(Stirling)公式为

n!2πn(ne)nn! \sim \sqrt{2\pi n}\left(\dfrac{n}{\e}\right)^n

limnn!ennn+12=2π\lim_{n \to \infty} \dfrac{n! \e^n}{n^{n+\frac{1}{2} }} = \sqrt{2 \pi}

实际上为

n!=2πn(ne)n(1+112n+1288n213951840n35712488320n4+)n! = \sqrt{2 \pi n}\left(\frac{n}{\e}\right)^{n}\left(1+\frac{1}{12 n}+\frac{1}{288 n^{2}}-\frac{139}{51840 n^{3}}-\frac{571}{2488320 n^{4}}+\cdots\right)

的截断形式。

任意项级数

交错级数

正负相间的级数,即形如

n=1(1)n+1un\sum_{n=1}^{\infty}(-1)^{n+1}u_n

的级数,其中 un>0u_n > 0,称为交错级数

莱布尼茨定理

若交错级数 n=1(1)n+1un\displaystyle \sum_{n=1}^{\infty}(-1)^{n+1} u_nun>0u_n > 0)满足

  1. un+1unu_{n+1} \le u_n
  2. limnun=0\lim\limits_{n \to \infty} u_n = 0

则交错级数 n=1(1)n+1un\displaystyle \sum_{n=1}^{\infty}(-1)^{n+1} u_n 收敛。余项 rnr_n 符号与余项第一项 (1)n+2un+1(-1)^{n+2}u_{n+1} 符号相同,且 rnun+1|r_n| \le u_{n+1}


莱布尼茨定理证明

考察偶数项部分和 {S2m}\left\lbrace S_{2m} \right\rbrace 与奇数项部分和 {S2m+1}\left\lbrace S_{2m+1} \right\rbrace。即

S2m=(u1u2)++(u2m1u2m)S2m+1=(u1u2)++(u2m1u2m)+u2m+1\begin{aligned} S_{2m} &= (u_1 - u_2) + \cdots + (u_{2m-1} - u_{2m})\\ S_{2m+1} &= (u_1 - u_2) + \cdots + (u_{2m-1} - u_{2m}) + u_{2m+1} \end{aligned}

由 1. 知 {S2m}\left\lbrace S_{2m} \right\rbrace 单调递增,而

0S2m=u1(u2u3)(u2m1u2m)u10 \le S_{2m} = u_1 - (u_2 - u_3) - \cdots - (u_{2m-1} - u_{2m}) \le u_1

由单调有界定理知 {S2m}\left\lbrace S_{2m} \right\rbrace 收敛,故 {S2m}\left\lbrace S_{2m} \right\rbrace 极限存在,设

limmS2m=S\lim_{m \to \infty} S_{2m} = S

S2m+1=S2m+u2m+1S_{2m+1} = S_{2m} + u_{2m+1}

由 2. 知 limmu2m+1=0\lim\limits_{m \to \infty} u_{2m+1} = 0,故

limmS2m+1=limmS2m+limmu2m+1=S\begin{aligned} \lim_{m \to \infty} S_{2m+1} &= \lim_{m \to \infty} S_{2m} + \lim_{m \to \infty} u_{2m+1}\\ &= S \end{aligned}

因此

limnS2m=limnS2m+1=S\lim_{n \to \infty} S_{2m} = \lim_{n \to \infty} S_{2m+1} = S

n=1(1)n+1un\displaystyle \sum_{n=1}^{\infty}(-1)^{n+1} u_n 收敛,且 0Su10\le S \le u_1

余项 rnr_n 可写作

rn=(1)n+2(un+1un+2+)r_n = (-1)^{n+2}(u_{n+1} - u_{n+2} + \cdots)

rn=un+1un+2+|r_n| = u_{n+1} - u_{n+2} + \cdots

也为交错级数,从而 rnun+1|r_n| \le u_{n+1}

绝对收敛与条件收敛

若级数 n=1un\displaystyle \sum_{n=1}^{\infty} u_n 的各项绝对值级数 n=1un\displaystyle \sum_{n=1}^{\infty} |u_n| 收敛,则称 n=1un\displaystyle \sum_{n=1}^{\infty} u_n 绝对收敛

若级数 n=1un\displaystyle \sum_{n=1}^{\infty} u_n 收敛,而 n=1un\displaystyle \sum_{n=1}^{\infty} |u_n| 发散,则称 n=1un\displaystyle \sum_{n=1}^{\infty} u_n 条件收敛

绝对收敛级数必定收敛。

证明

n=1un\displaystyle \sum_{n=1}^{\infty} u_n 绝对收敛,即 n=1un\displaystyle \sum_{n=1}^{\infty} |u_n| 收敛。

vn=12(un+un)v_n = \dfrac{1}{2}(|u_n| + u_n)

0vnun0 \le v_n \le |u_n|,比较判别法有 n=1vn\displaystyle \sum_{n=1}^{\infty} v_n 收敛。从而 n=12vn\displaystyle \sum_{n=1}^{\infty} 2 v_n 收敛。

un=2vnunu_n = 2v_n - |u_n|,收敛级数的基本性质有

n=1un=n=12vnn=1un\sum_{n=1}^{\infty} u_n = \sum_{n=1}^{\infty} 2v_n - \sum_{n=1}^{\infty} |u_n|

从而 n=1un\displaystyle \sum_{n=1}^{\infty} u_n 收敛。

类似地令

wn=12(unun)w_n = \dfrac{1}{2}(|u_n| - u_n)

  • 若级数 n=1un\displaystyle \sum_{n=1}^{\infty} u_n 绝对收敛,则 n=1vn\displaystyle \sum_{n=1}^{\infty}v_nn=1wn\displaystyle \sum_{n=1}^{\infty}w_n 均收敛,且 n=1un=n=1vnn=1wn\displaystyle \sum_{n=1}^{\infty}u_n = \sum_{n=1}^{\infty}v_n - \sum_{n=1}^{\infty}w_n
  • 若级数 n=1un\displaystyle \sum_{n=1}^{\infty} u_n 条件收敛,则 n=1vn\displaystyle \sum_{n=1}^{\infty}v_nn=1wn\displaystyle \sum_{n=1}^{\infty}w_n 均发散。

因此绝对值级数 n=1un\displaystyle \sum_{n=1}^{\infty}|u_n| 发散不能断定级数 n=1un\displaystyle \sum_{n=1}^{\infty}u_n 发散。

但若应用达朗贝尔判别法和柯西判别法时,绝对值级数发散,则原级数必发散。

绝对收敛级数 n=1un\displaystyle\sum_{n=1}^{\infty} u_n更序级数(即将级数项重新排列)n=1un\displaystyle\sum_{n=1}^{\infty} u'_n 仍为绝对收敛级数,且 n=1un=n=1un\displaystyle\sum_{n=1}^{\infty} u'_n = \displaystyle\sum_{n=1}^{\infty} u_n

证明

先证明 n=1un\displaystyle\sum_{n=1}^{\infty} u_n 为收敛的正项级数。

考虑更序级数 n=1un\displaystyle\sum_{n=1}^{\infty} u'_n 部分和 SkS_k',因为

u1=un1,,uk=unku_1' = u_{n_1},\, \cdots,\, u_k' = u_{n_k}

nmax{n1,,nk}n \ge \max \left\lbrace n_1, \cdots, n_{k} \right\rbrace,则有

Sk=u1++uk=un1++unku1++un=Sn\begin{aligned} S_{k}' &= u_1' + \cdots + u_k'\\ &= u_{n_1} + \cdots + u_{n_k}\\ &\le u_1 + \cdots + u_n\\ &= S_n \end{aligned}

对于正项级数 n=1un\displaystyle\sum_{n=1}^{\infty} u_n 的和 SS,有 SnSS_n \le S,于是对一切 kkSkSS_k' \le S,从而 n=1un\displaystyle\sum_{n=1}^{\infty} u'_n 收敛。

设其和为 SS',则 SSS' \le S

n=1un\displaystyle\sum_{n=1}^{\infty} u_n 也可以看作是 n=1un\displaystyle\sum_{n=1}^{\infty} u'_n 的更序级数,故 SSS \le S'。从而 S=SS = S'

再证明 n=1un\displaystyle\sum_{n=1}^{\infty} u_n 为任意绝对收敛级数的情形。

{vn=12(un+un)wn=12(unun)\begin{cases} v_n = \dfrac{1}{2} (|u_n| + u_n)\\ w_n = \dfrac{1}{2} (|u_n| - u_n) \end{cases}

显然有 0vn,wnun0 \le v_n, w_n \le |u_n|

因为 n=1un\displaystyle\sum_{n=1}^{\infty} |u_n| 收敛,则级数 n=1vn\displaystyle\sum_{n=1}^{\infty} v_n 和级数 n=1wn\displaystyle\sum_{n=1}^{\infty} w_n 收敛。不妨设 n=1vn=V,n=1wn=W\displaystyle\sum_{n=1}^{\infty} v_n = V, \displaystyle\sum_{n=1}^{\infty} w_n = W

因为 un=vnwn,un=vn+wnu_n = v_n - w_n, |u_n| = v_n + w_n,则

n=1un=VW,n=1un=V+W\displaystyle\sum_{n=1}^{\infty} u_n = V - W,\quad \displaystyle\sum_{n=1}^{\infty} |u_n| = V + W

由上面证明过的情形有,n=1un\displaystyle\sum_{n=1}^{\infty} |u_n| 的更序级数 n=1un\displaystyle\sum_{n=1}^{\infty} |u'_n| 成立

n=1un=V+W\displaystyle\sum_{n=1}^{\infty} |u'_n| = V + W

即更序级数 n=1un\displaystyle\sum_{n=1}^{\infty} u'_n 绝对收敛。

再设 n=1vn\displaystyle\sum_{n=1}^{\infty} v'_nn=1wn\displaystyle\sum_{n=1}^{\infty} w'_n 分别为 n=1vn\displaystyle\sum_{n=1}^{\infty} v_nn=1wn\displaystyle\sum_{n=1}^{\infty} w_n 的更序级数,则同理有

n=1vn=V,n=1wn=W\displaystyle\sum_{n=1}^{\infty} v'_n = V,\quad \displaystyle\sum_{n=1}^{\infty} w'_n = W

un=vnwnu_n' = v_n' - w_n',所以

n=1un=n=1(vnwn)=VW=n=1un\begin{aligned} \displaystyle\sum_{n=1}^{\infty} u'_n &= \displaystyle\sum_{n=1}^{\infty} (v'_n-w'_n)\\ &= V - W\\ &= \displaystyle\sum_{n=1}^{\infty} u_n \end{aligned}

得证。

黎曼重排定理

对于条件收敛级数 n=1un\displaystyle\sum_{n=1}^{\infty} u_n,可以重新排列其顺序,使得新的级数 n=1un\displaystyle\sum_{n=1}^{\infty} u'_n 收敛到任意给定的值,或者发散。

阿贝尔(Abel)变换

Bn=k=1nbkB_n = \displaystyle \sum_{k=1}^{n}b_k,则

k=nmakbk=k=nmak(BkBk1)=k=nmakBkk=nmakBk1=k=nmakBkk=n1m1ak+1Bk=k=nm(akak+1)Bk+am+1BmanBn1\begin{aligned} \sum_{k=n}^{m}a_{k}b_{k} &= \sum_{k=n}^{m}a_{k}(B_{k} - B_{k-1})\\ &= \sum_{k=n}^{m}a_{k}B_{k} - \sum_{k=n}^{m}a_{k}B_{k-1}\\ &= \sum_{k=n}^{m}a_{k}B_{k} - \sum_{k=n-1}^{m-1}a_{k+1}B_{k}\\ &= \sum_{k=n}^{m}(a_{k}-a_{k+1})B_{k} + a_{m+1}B_{m} - a_{n}B_{n-1} \end{aligned}


记前向差分算子 Δ\DeltaΔan=an+1an\Delta a_{n} = a_{n+1} - a_{n},则有阿贝尔变换

k=nmakΔbk=am+1bm+1anbnk=nmbk+1Δak\begin{aligned} \sum_{k=n}^{m}a_{k}\Delta b_{k} = a_{m+1}b_{m+1} - a_n b_n - \sum_{k=n}^{m} b_{k+1} \Delta a_{k} \end{aligned}

阿贝尔判别法

  1. 级数 n=1bn\displaystyle\sum_{n=1}^{\infty} b_n 收敛
  2. 数列 {an}\left\lbrace a_n \right\rbrace 单调有界

则级数 n=1anbn\displaystyle\sum_{n=1}^{\infty} a_n b_n 收敛。

证明

级数 n=1bn\displaystyle\sum_{n=1}^{\infty} b_n 收敛得,存在 MM 使得 BnM|B_n| \le M。同时 {an}\left\lbrace a_n \right\rbrace 单调得,akak+1a_{k} - a_{k+1} 同号。

k=nmakbk=k=nm(akak+1)Bk+am+1BmanBn1k=nmakak+1Bk+am+1Bm+anBn1M(k=nmakak+1+am+1+an)=M(anam+1+am+1+an)\begin{aligned} \left\lvert \sum_{k=n}^{m}a_{k}b_{k} \right\rvert &= \left\lvert \sum_{k=n}^{m}(a_{k}-a_{k+1})B_{k} + a_{m+1}B_{m} - a_{n}B_{n-1} \right\rvert\\ &\le \sum_{k=n}^{m}\left\lvert a_{k}-a_{k+1} \right\rvert \left\lvert B_{k} \right\rvert + \left\lvert a_{m+1}B_{m} \right\rvert + \left\lvert a_{n}B_{n-1} \right\rvert\\ &\le M \left(\sum_{k=n}^{m}\left\lvert a_{k}-a_{k+1} \right\rvert + \left\lvert a_{m+1} \right\rvert + \left\lvert a_n \right\rvert\right)\\ &= M\left(\left\lvert a_{n} - a_{m+1} \right\rvert + \left\lvert a_{m+1} \right\rvert + \left\lvert a_n \right\rvert\right) \end{aligned}

{an}\left\lbrace a_n\right\rbrace 有界,故 n=1anbn\displaystyle\sum_{n=1}^{\infty} a_nb_n 任意部分和有界,从而 n=1anbn\displaystyle\sum_{n=1}^{\infty} a_n b_n 收敛。

狄利克雷判别法

  1. 级数 n=1bn\displaystyle\sum_{n=1}^{\infty} b_n 部分和数列有界
  2. 数列 {an}\left\lbrace a_n \right\rbrace 单调趋于零

则级数 n=1anbn\displaystyle\sum_{n=1}^{\infty} a_n b_n 收敛。

柯西乘积

若级数 n=1an\displaystyle\sum_{n=1}^{\infty} a_nn=1bn\displaystyle\sum_{n=1}^{\infty} b_n 收敛,则称 n=1cn\displaystyle\sum_{n=1}^{\infty} c_n 为级数 n=1an\displaystyle\sum_{n=1}^{\infty} a_nn=1bn\displaystyle\sum_{n=1}^{\infty} b_n柯西乘积,其中

cn=k=1nakbnk+1\begin{aligned} c_n = \sum_{k=1}^{n}a_{k}b_{n-k+1} \end{aligned}

即按对角线求和。

柯西定理

若级数 n=1un\displaystyle\sum_{n=1}^{\infty} u_nn=1vn\displaystyle\sum_{n=1}^{\infty} v_n 绝对收敛,和分别为 s,σs, \sigma,则它们各项之积 uivju_iv_j 按任何方式排列所构成的级数绝对收敛,且和为 sσs\sigma

证明

w1,,wn,w_1, \cdots, w_n, \cdots 表示按某一种次序排列 uivju_i v_j 所构成的一个数列,考虑级数

w1+wn+|w_1| + \cdots |w_n| + \cdots

sns_n^{*} 是它的部分和

sn=k=1nwk=k=1nunkvmks_n^{*} = \sum_{k=1}^{n}|w_k| = \sum_{k=1}^{n} |u_{n_k}v_{m_k}|

μ=max{n1,,nn,m1,,mn}\mu = \max \left\lbrace n_1, \cdots, n_n, m_1, \cdots, m_n \right\rbrace

又记

Uμ=k=1μukVμ=k=1μvkU_{\mu}^{*} = \sum_{k=1}^{\mu}|u_{k}|\\ V_{\mu}^{*} = \sum_{k=1}^{\mu}|v_{k}|

n=1un\displaystyle\sum_{n=1}^{\infty} u_nn=1vn\displaystyle\sum_{n=1}^{\infty} v_n 绝对收敛,设 U=n=1un,V=n=1vnU^{*} = \displaystyle\sum_{n=1}^{\infty} |u_n|, V^{*} = \displaystyle\sum_{n=1}^{\infty} |v_n|,则有 UμU,VμVμU_{\mu}^{*} \le U^{*}, V_{\mu}^{*} \le V^{*}_{\mu},从而有

sn=k=1nunkvmkk=1nunkvmkk=1nunkk=1nvmkk=1μukk=1μvkUμVμUV\begin{aligned} s_n^{*} &= \sum_{k=1}^{n} |u_{n_k}v_{m_k}|\\ &\le \sum_{k=1}^{n} |u_{n_k}||v_{m_k}|\\ &\le \sum_{k=1}^{n} |u_{n_k}| \sum_{k=1}^{n} |v_{m_k}|\\ &\le \sum_{k=1}^{\mu} |u_{k}| \sum_{k=1}^{\mu} |v_{k}|\\ &\le U_{\mu}^{*}V_{\mu}^{*}\\ &\le U^{*}V^{*} \end{aligned}

由整项级数收敛基本定理,n=1wn\displaystyle\sum_{n=1}^{\infty} |w_n| 收敛,即 n=1wn\displaystyle\sum_{n=1}^{\infty} w_n 绝对收敛,因此其更序级数绝对收敛。

为方便计算,按正方形法排列所构成的级数进行求和,即

n=1wn=u1v1+(u1v2+u2v2+u2v1)+\begin{aligned} \displaystyle\sum_{n=1}^{\infty} w'_n &= u_1 v_1 + (u_1 v_2 + u_2 v_2 + u_2 v_1) + \cdots \end{aligned}

n=1wn\displaystyle\sum_{n=1}^{\infty} w'_n 部分和为 WnW_n,则 Wn=snσnW_n = s_n \sigma_n,于是

n=1wn=limnWn=limnsnσn=sσ\begin{aligned} \displaystyle\sum_{n=1}^{\infty} w'_n &= \lim\limits_{n \to \infty} W_n\\ &= \lim\limits_{n \to \infty} s_n \sigma_n\\ &= s\sigma \end{aligned}

从而得证。

函数项级数

给定定义在区间 II 的函数列

u1(x),,un(x),u_1(x), \cdots, u_n(x), \cdots

则形式和

n=1un(x)\displaystyle\sum_{n=1}^{\infty} u_n(x)

称为定义在区间 II 上的函数项级数

对每一个确定的值 x0Ix_0 \in I,函数项级数成为常数项级数,若常数项级数

n=1un(x0)\displaystyle\sum_{n=1}^{\infty} u_n(x_0)

收敛,则称函数项级数x0x_0 处收敛,点 x0x_0 称为函数项级数的收敛点。否则称函数项级数在 x0x_0 处发散,点 x0x_0 称为函数项级数的发散点

收敛点的全体称为函数项级数的收敛域,发散点的全体称为函数项级数的发散域

对于收敛域内任一点 xx,函数项级数成为一收敛的常数项级数,故可定义函数项级数的和函数 S(x)S(x),即

S(x)=n=1un(x)S(x) = \displaystyle\sum_{n=1}^{\infty} u_n(x)

及函数项级数前 nn 项的部分和为 Sn(x)S_n(x),收敛域上有

limnSn(x)=S(x)\lim\limits_{n \to \infty}S_n(x) = S(x)

此时称

rn(x)=S(x)Sn(x)=k=n+1uk(x)r_n(x) = S(x) - S_n(x) = \displaystyle\sum_{k=n+1}^{\infty} u_k(x)

为函数项级数的余项。并有 limnrn(x)=0\lim\limits_{n \to \infty} r_n(x) = 0

对函数项级数 S(x)=n=1un(x)S(x) = \displaystyle\sum_{n=1}^{\infty} u_n(x),且任意 un(x)u_n(x) 连续,不一定有下面的结论成立

  • S(x)S(x) 连续(如 un(x)=xnxn1u_n(x) = x^n - x^{n-1}(1,1](-1, 1] 上不成立)
  • S(x)=n=1un(x)S'(x) = \displaystyle\sum_{n=1}^{\infty} u'_n(x)
  • S(x) ⁣dx=n=1un(x) ⁣dx\displaystyle \int S(x) \d x = \sum_{n=1}^{\infty} \int u_n(x) \d x

上面的问题在于两个及以上极限过程的顺序问题,即两个极限何时可以交换。

一致收敛

设函数项级数

u1(x)++un(x)+u_{1}(x)+\cdots+u_{n}(x)+\cdots

在区间 II 上收敛于和函数 S(x)S(x)

即对于区间 II 上的每一个值 x0x_{0},数项级数 n=1un(x0)\displaystyle \sum_{n=1}^{\infty} u_{n}\left(x_{0}\right) 收敛于 S(x0)S\left(x_{0}\right),即部分和数列 Sn(x0)=i=1nui(x0)S_{n}\left(x_{0}\right)=\displaystyle \sum_{i=1}^{n} u_{i}\left(x_{0}\right) 收敛于 S(x0)(n)S\left(x_{0}\right)(n \to \infty)

则对于任意给定的 ε>0\varepsilon>0 以及区间 II 上的每一个固定点 x0x_{0},都存在一个正整数 NN,使得当 n>Nn>N 时,有

S(x0)Sn(x0)<ε\left|S\left(x_{0}\right)-S_{n}\left(x_{0}\right)\right|<\varepsilon

rn(x0)=i=n+1ui(x0)<ε\left|r_{n}\left(x_{0}\right)\right|=\left|\sum_{i=n+1}^{\infty} u_{i}\left(x_{0}\right)\right|<\varepsilon

这里的 NN 一般不仅依赖于 ε\varepsilon,而且还依赖于 x0x_{0},记为 N(x0,ε)N(x_0, \varepsilon)。这种收敛即为逐点收敛

若对某一函数项级数,都能有一个这样的正整数 NN,使得其仅依赖于 ε\varepsilon 而不依赖于 x0x_0,即对于 II 上任意一点 x0x_0 都有 N=N(ε)N = N(\varepsilon),则称该函数项级数在区间 II 上一致收敛。

柯西一致收敛准则

设函数项级数 n=1un(x)\displaystyle\sum_{n=1}^{\infty} u_n(x),若对任意给定的 ε>0\varepsilon > 0,都存在一个正整数 NN,使得当 n>Nn > N 时,对于区间 II 上的任意一点 xx 都有

rn(x)=S(x)Sn(x)<ε\left|r_n(x)\right| = \left|S(x) - S_n(x)\right| < \varepsilon

则称函数项级数 n=1un(x)\displaystyle\sum_{n=1}^{\infty} u_n(x) 在区间 II一致收敛S(n)S(n),也称 {Sn(x)}\left\lbrace S_n(x) \right\rbrace 在区间 II 上一致收敛于 S(x)S(x)。记作 n=1un(x)S(x)\displaystyle\sum_{n=1}^{\infty} u_n(x) \rightrightarrows S(x)


几何意义:n>N(ε)n > N(\varepsilon) 时,区间 II 上所有曲线 y=Sn(x)y = S_n(x) 都将位于曲线

y1=S(x)+εy2=S(x)εy_1 = S(x) + \varepsilon\\ y_2 = S(x) - \varepsilon

GGB 重出江湖。

下面三个函数项级数均逐点收敛到 00

fn(x)=xenxf_n(x) = x \e^{-nx} 一致收敛(峰值往左,高度下降)。

fn(x)=nxenxf_n(x) = nx \e^{-nx} 不一致收敛,但一致有界(峰值往左,但高度不变)。

fn(x)=n2xenxf_n(x) = n^2 x \e^{nx} 不一致收敛,也不一致有界(峰值往左,高度也上升)。

绝对一致收敛

n=1un(x)\displaystyle\sum_{n=1}^{\infty} |u_n(x)| 在区间 II 上一致收敛,则称函数项级数 n=1un(x)\displaystyle\sum_{n=1}^{\infty} u_n(x) 在区间 II绝对一致收敛

绝对一致收敛的函数项级数一定是一致收敛的,但反之不一定成立。

魏尔斯特拉斯判别法(强级数判别法)

设函数项级数 n=1un(x)\displaystyle\sum_{n=1}^{\infty} u_n(x) 在区间 II 上,对于任意 xIx \in I,都有 un(x)an\left|u_n(x)\right| \le a_n,且 n=1an\displaystyle\sum_{n=1}^{\infty} a_n 收敛,则函数项级数 n=1un(x)\displaystyle\sum_{n=1}^{\infty} u_n(x) 在区间 II 上一致收敛。

其中的 n=1an\displaystyle\sum_{n=1}^{\infty} a_n 称为函数项级数 n=1un(x)\displaystyle\sum_{n=1}^{\infty} u_n(x)强级数

证明

n=1an\displaystyle\sum_{n=1}^{\infty} a_n 收敛即 n=1an\displaystyle\sum_{n=1}^{\infty} a_n 任意部分和有界。即 ε>0\forall \varepsilon > 0,都有 NN 使得 k=n+1mak\left\lvert \displaystyle \sum_{k=n+1}^{m}a_{k} \right\rvert 对任意 m>n>Nm > n > N 成立。

uk(x)ak\left\lvert u_{k}(x) \right\rvert \le a_{k} 对任意 xIx \in I 成立,所以

k=n+1muk(x)k=n+1muk(x)k=n+1mak<ε\begin{aligned} \left\lvert \sum_{k=n+1}^{m} u_{k}(x) \right\rvert &\le \sum_{k=n+1}^{m} \left\lvert u_{k}(x) \right\rvert\\ &\le \sum_{k=n+1}^{m} a_{k}\\ &< \varepsilon \end{aligned}

进而 k=n+1uk(x)<ε\left\lvert \displaystyle \sum_{k=n+1}^{\infty}u_{k}(x) \right\rvert < \varepsilon 对任意 xI,nN+x \in I, n \in \N^{+} 成立,即

rn(x)=S(x)Sn(x)=k=n+1uk(x)<ε\begin{aligned} \left\lvert r_n(x) \right\rvert &= \left\lvert S(x) - S_n(x) \right\rvert\\ &= \left\lvert \sum_{k=n+1}^{\infty} u_{k}(x) \right\rvert\\ &< \varepsilon \end{aligned}

所以函数项级数 n=1un(x)\displaystyle\sum_{n=1}^{\infty} u_n(x) 在区间 II 上一致收敛。

若函数项级数 n=1un(x)\displaystyle\sum_{n=1}^{\infty} u_n(x)[a,b][a, b] 上一致连续,且每一项 un(x)u_n(x)[a,b][a, b] 上连续,则其和函数 S(x)=n=1un(x)S(x) = \displaystyle\sum_{n=1}^{\infty} u_n(x)[a,b][a, b] 上也连续。

证明

n=1un(x)S(x)\displaystyle\sum_{n=1}^{\infty} u_n(x) \rightrightarrows S(x),对任意 ε>0\varepsilon > 0,存在 N=N(ε)N = N(\varepsilon) 使得 n>Nn > N 时,对任意 x[a,b]x \in [a, b]

S(x)Sn(x)<rn(x)<ε3\left\lvert S(x) - S_n(x) \right\rvert < r_n(x) < \dfrac{\varepsilon}{3}

故对 [a,b][a, b] 上一个固定点 x0x_0,也有 rn(x0)<ε3r_n(x_0) < \dfrac{\varepsilon}{3}

对选定的 NNSN+1(x)S_{N+1}(x) 也为在 x0x_0 连续的函数,故对上面的 ε>0\varepsilon > 0,存在 δ>0\delta > 0 使得当 xx0<δ\left\lvert x - x_0 \right\rvert < \delta 时,有

SN+1(x)SN+1(x0)<ε3\left\lvert S_{N+1}(x) - S_{N+1}(x_0) \right\rvert < \dfrac{\varepsilon}{3}

因此当 xx0<δ\left\lvert x - x_0 \right\rvert < \delta 时,有

S(x)S(x0)S(x)SN+1(x)+SN+1(x)SN+1(x0)+SN+1(x0)S(x0)<ε3+ε3+ε3=ε\begin{aligned} \left\lvert S(x) - S(x_0) \right\rvert &\le \left\lvert S(x) - S_{N+1}(x) \right\rvert + \left\lvert S_{N+1}(x) - S_{N+1}(x_0) \right\rvert + \left\lvert S_{N+1}(x_0) - S(x_0) \right\rvert\\ &< \dfrac{\varepsilon}{3} + \dfrac{\varepsilon}{3} + \dfrac{\varepsilon}{3}\\ &= \varepsilon \end{aligned}

逐项积分

如果级数 n=1un(x)\displaystyle\sum_{n=1}^{\infty} u_n(x) 的各项 un(x)u_n(x) 在区间 [a,b][a, b] 连续,且 n=1un(x)\displaystyle\sum_{n=1}^{\infty} u_n(x)[a,b][a, b] 上一致收敛于 S(x)S(x),则级数 n=1un(x)\displaystyle\sum_{n=1}^{\infty} u_n(x)[a,b][a, b] 上可以逐项积分,即

abS(x) ⁣dx=n=1abun(x) ⁣dx\int_a^b S(x) \d x = \sum_{n=1}^{\infty} \int_a^b u_n(x) \d x

证明

Sn(x)S(x)S_n(x) \rightrightarrows S(x) 得任意 ε>0\varepsilon > 0,存在 NN 使得 n>Nn > N 时,对任意 x[a,b]x \in [a, b]S(x)Sn(x)<εba\left\lvert S(x) - S_n(x) \right\rvert < \dfrac{\varepsilon}{b - a}

abS(x) ⁣dxk=1nuk(x) ⁣dx=ab(S(x)k=1nuk(x)) ⁣dx=ab(S(x)Sn(x)) ⁣dxabS(x)Sn(x) ⁣dxabεba ⁣dx=ε\begin{aligned} \left\lvert \int_a^b S(x) \d x - \sum_{k=1}^{n}u_{k}(x) \d x \right\rvert &= \left\lvert \int_a^b \left(S(x) - \sum_{k=1}^{n}u_k(x)\right) \d x \right\rvert\\ &= \left\lvert \int_a^b \left(S(x) - S_n(x)\right) \d x \right\rvert\\ &\le \int_a^b \left\lvert S(x) - S_n(x)\right\rvert \d x \\ &\le \int_a^b \dfrac{\varepsilon}{b - a} \d x\\ &= \varepsilon \end{aligned}

逐项求导

如果级数 n=1un(x)\displaystyle\sum_{n=1}^{\infty} u_n(x) 在区间 [a,b][a, b] 上一致收敛于 S(x)S(x)un(x)u_n(x) 的导函数 un(x)u'_n(x)[a,b][a, b] 上连续,并且级数 n=1un(x)\displaystyle\sum_{n=1}^{\infty} u'_n(x)[a,b][a, b] 上一致连续,则和函数 S(x)S(x) 在区间 [a,b][a, b] 上可导,且可逐项求导,即

S(x)=n=1un(x)S'(x) = \sum_{n=1}^{\infty} u'_n(x)

并且 S(x)S'(x) 也在 [a,b][a, b] 上连续。

证明

n=1un(x)φ(x)\displaystyle\sum_{n=1}^{\infty} u'_n(x) \rightrightarrows \varphi(x),即

φ(x)=n=1un(x)\varphi(x) = \sum_{n=1}^{\infty} u'_n(x)

由逐项积分,取 x[a,b]x \in [a, b],有

axφ(t) ⁣dt=axn=1un(t) ⁣dt=n=1axun(t) ⁣dt=n=1(un(x)un(a))=n=1un(x)n=1un(a)=S(x)S(a)\begin{aligned} \int_a^x \varphi(t) \d t &= \int_a^x \sum_{n=1}^{\infty} u'_n(t) \d t\\ &= \sum_{n=1}^{\infty} \int_a^x u'_n(t) \d t\\ &= \sum_{n=1}^{\infty} \left(u_n(x) - u_n(a)\right)\\ &= \sum_{n=1}^{\infty} u_n(x) - \sum_{n=1}^{\infty} u_n(a)\\ &= S(x) - S(a) \end{aligned}

再由 un(x)u'_n(x) 的一致连续性有 φ(x)\varphi(x)[a,b][a, b] 上连续,所以

S(x)=S(a)+axφ(t) ⁣dtS(x) = S(a) + \int_a^x \varphi(t) \d t

可导,且 S(x)=φ(x)=n=1un(x)S'(x) = \varphi(x) = \displaystyle\sum_{n=1}^{\infty} u'_n(x)

幂级数

本节幂级数下标基本从 n=0n=0 开始,与上面有所区别。

幂级数

形如

n=0an(xx0)n=a0+a1(xx0)+a2(xx0)2+\displaystyle\sum_{n=0}^{\infty} a_n(x - x_0)^n = a_0 + a_1(x - x_0) + a_2(x - x_0)^2 + \cdots

的函数项级数称为幂级数

为简单起见,讨论幂级数时,一般取 x0=0x_0 = 0,即

n=0anxn=a0+a1x+a2x2+\displaystyle\sum_{n=0}^{\infty} a_n x^n = a_0 + a_1 x + a_2 x^2 + \cdots

阿贝尔定理

对于幂级数 n=0anxn\displaystyle\sum_{n=0}^{\infty} a_nx^n,有

  1. 若此级数在 x10x_1 \ne 0 处收敛,则对任意满足 x<x1|x| < |x_1|xx,都有 n=0anxn\displaystyle\sum_{n=0}^{\infty} a_nx^nxx 处绝对收敛。
  2. 若此级数在 x20x_2 \ne 0 处发散,则对任意满足 x>x2|x| > |x_2|xx,都有 n=0anxn\displaystyle\sum_{n=0}^{\infty} a_nx^nxx 处发散。

即幂级数收敛域除端点外是关于 x=0x = 0 对称的。

证明

既然 n=0anx1n\displaystyle\sum_{n=0}^{\infty} a_nx_1^n 收敛,所以 limnanx1n=0\lim\limits_{n \to \infty} a_nx_1^n = 0,从而数列 {anx1n}\left\lbrace a_nx_1^n \right\rbrace 有界,即存在 M>0M > 0 使

anx1nM\left\lvert a_n x_1^n \right\rvert \le M

于是对任意一个满足 x<x1|x| < |x_1|xx,有

anxn=anx1nxx1nMxx1n\begin{aligned} \left\lvert a_n x^n \right\rvert &= \left\lvert a_n x_1^n \right\rvert \left\lvert \dfrac{x}{x_1} \right\rvert^n\\ &\le M \left\lvert \dfrac{x}{x_1} \right\rvert^n \end{aligned}

xx1<1\left\lvert \dfrac{x}{x_1} \right\rvert < 1,因此等比数列 n=0xx1n\displaystyle\sum_{n=0}^{\infty} \left\lvert \dfrac{x}{x_1} \right\rvert^n 收敛。

比较判别法知 n=0anxn\displaystyle\sum_{n=0}^{\infty} \left\lvert a_nx^n \right\rvert 收敛,即 n=0anxn\displaystyle\sum_{n=0}^{\infty} a_nx^n 绝对收敛。

第二个结论可由第一个结论推出:反证法,设 x>x2|x| > |x_2| 时级数收敛,由第一个结论知 x2<x|x_2| < |x|,级数在 x2x_2 处收敛,矛盾。

柯西-阿达马定理/阿贝尔第一定理(Cauchy-Hadamard/Abel)

对幂级数 n=0anxn\displaystyle\sum_{n=0}^{\infty} a_nx^n,存在 0R+0 \le R \le +\infty ,使得

  1. x<R|x| < R 时,级数绝对收敛;
  2. x>R|x| > R 时,级数发散;
  3. x=R|x| = R 时,级数可能收敛,可能发散。

RR 称为幂级数的收敛半径,开区间 (R,R)(-R, R) 称为幂级数的收敛区间,所有收敛点的集合称为幂级数的收敛域

也就是说收敛区间与收敛域未必相同。

达朗贝尔判别法(D'Alembert)

对于幂级数 n=0anxn\displaystyle\sum_{n=0}^{\infty} a_n x^n,若

limnan+1an=l\lim\limits_{n \to \infty} \left\lvert \dfrac{a_{n+1}}{a_n} \right\rvert = l

则级数的收敛半径

R={1l0<l<+0l=++l=0R = \begin{cases} \dfrac{1}{l} & 0 < l < + \infty\\ 0 & l = +\infty\\ +\infty & l = 0 \end{cases}

证明

x=0x = 0 时显然,不妨设 x0x \ne 0,则有

limnan+1xn+1anxn=limnan+1anx={lx0<l<+0l=0+l=+\lim_{n\to \infty }\left\lvert \dfrac{a_{n+1}x^{n+1}}{a_nx^n} \right\rvert = \lim_{n\to \infty }\left\lvert \dfrac{a_{n+1}}{a_n} \right\rvert |x| = \begin{cases} l|x| & 0 < l < +\infty\\ 0 & l = 0\\ +\infty & l = +\infty \end{cases}

0<l<+0 < l < + \infty ,当 x<1l|x| < \dfrac{1}{l} 时,级数 n=0anxn\displaystyle\sum_{n=0}^{\infty} a_n x^n 绝对收敛,当 x>1l|x| > \dfrac{1}{l} 时,级数发散。故 R=1lR = \dfrac{1}{l}

l=0l = 0,则级数 n=0anxn\displaystyle\sum_{n=0}^{\infty} a_n x^n 对任意 xx 都绝对收敛,故 R=+R = +\infty

l=+l = +\infty,则级数 n=0anxn\displaystyle\sum_{n=0}^{\infty} a_n x^n 对任意 xx 都发散,故 R=0R = 0

柯西判别法

对于幂级数 n=0anxn\displaystyle\sum_{n=0}^{\infty} a_n x^n,若

limnann=l\lim\limits_{n \to \infty} \sqrt[n]{\left\lvert a_n \right\rvert} = l

则级数的收敛半径

R={1l0<l<+0l=++l=0R = \begin{cases} \dfrac{1}{l} & 0 < l < + \infty\\ 0 & l = +\infty\\ +\infty & l = 0 \end{cases}

设幂级数 n=0anxn\displaystyle\sum_{n=0}^{\infty} a_nx^nn=0bnxn\displaystyle\sum_{n=0}^{\infty} b_nx^n 收敛半径分别为 R1,R2R_1, R_2,令 R=min{R1,R2}R = \min\left\lbrace R_1, R_2 \right\rbrace,则对于 x<R|x| < R,有

  1. n=0(an±bn)xn=n=0anxn±n=0bnxn\displaystyle\sum_{n=0}^{\infty} (a_n \pm b_n)x^n = \displaystyle\sum_{n=0}^{\infty} a_nx^n \pm \displaystyle\sum_{n=0}^{\infty} b_nx^n
  2. n=0cnxn=(n=0anxn)(n=0bnxn)\displaystyle\sum_{n=0}^{\infty} c_nx^n = \left(\displaystyle\sum_{n=0}^{\infty} a_nx^n\right) \cdot \left(\displaystyle\sum_{n=0}^{\infty} b_nx^n\right)
    • 其中 cn=a0bn+a1bn1++anb0c_n = a_0b_n + a_1b_{n-1} + \cdots + a_nb_0
  3. b00b_0 \ne 0 时,在 x=0x = 0 的适当邻域内,两幂级数可以相除,即 n=0cnxn=n=0anxnn=0bnxn\displaystyle\sum_{n=0}^{\infty} c_nx^n = \dfrac{\displaystyle\sum_{n=0}^{\infty} a_nx^n}{\displaystyle\sum_{n=0}^{\infty} b_nx^n}
    • 系数可由 n=0anxn=(n=0bnxn)(n=0cnxn)\displaystyle\sum_{n=0}^{\infty} a_nx^n = \left(\displaystyle\sum_{n=0}^{\infty} b_nx^n\right) \cdot \left(\displaystyle\sum_{n=0}^{\infty} c_nx^n\right) 依次确定。
    • 收敛区间可能比原来两级数的收敛半径

内闭一致收敛性(阿贝尔第二定理)

设幂级数 n=0anxn\displaystyle\sum_{n=0}^{\infty} a_nx^n 收敛半径 R>0R > 0,收敛域为 XX,任取闭区间 [a,b]X[a, b] \subseteq X,则有该幂级数在 [a,b][a, b] 上一致收敛。

幂级数 n=0anxn\displaystyle\sum_{n=0}^{\infty} a_nx^n 的和函数 S(x)S(x) 在其收敛域 II 上连续。

特别地,若 n=0anxn\displaystyle\sum_{n=0}^{\infty} a_nx^nRR 处收敛,则 S(x)S(x)RR 处左连续,若其在 R-R 处收敛,则 S(x)S(x)R-R 处右连续。

幂级数 n=0anxn\displaystyle\sum_{n=0}^{\infty} a_nx^n 的和函数 S(x)S(x) 在其收敛域 II 上可逐项积分,且有逐项积分公式

0xS(t) ⁣dt=0x(n=0antn)=n=00xantn ⁣dt=n=0ann+1xn+1\begin{aligned} \int_0^x S(t) \d t &= \int_0^x \left( \displaystyle\sum_{n=0}^{\infty} a_nt^n \right) \\ &= \displaystyle\sum_{n=0}^{\infty} \int_0^x a_nt^n \d t\\ &= \displaystyle\sum_{n=0}^{\infty} \dfrac{a_n}{n+1} x^{n+1} \end{aligned}

幂级数 n=0anxn\displaystyle\sum_{n=0}^{\infty} a_nx^n 的和函数 S(x)S(x) 在其收敛域 II 上可逐项求导,且有逐项求导公式

S(x)=(n=0anxn)=n=0(anxn)=n=1nanxn1\begin{aligned} S'(x) &= \left( \displaystyle\sum_{n=0}^{\infty} a_nx^n \right)'\\ &= \displaystyle\sum_{n=0}^{\infty} \left( a_nx^n \right)'\\ &= \displaystyle\sum_{n=1}^{\infty} na_nx^{n-1} \end{aligned}

求幂级数 n=11nxn\displaystyle\sum_{n=1}^{\infty} \dfrac{1}{n} x^n,并计算 n=1(1)n1n\displaystyle\sum_{n=1}^{\infty} \dfrac{(-1)^{n-1}}{n}


考虑具体一个例子,来看看过程。

过程

l=limnan+1an=limn1n+1n1=1\begin{aligned} l &= \lim\limits_{n \to \infty} \left| \dfrac{a_{n+1}}{a_n} \right| \\ &= \lim\limits_{n \to \infty} \left| \dfrac{1}{n+1} \cdot \dfrac{n}{1} \right| \\ &= 1 \end{aligned}

从而收敛半径 R=1l=1R = \dfrac{1}{l} = 1

x=1x = 1 时,幂级数为调和级数,发散;x=1x = -1 时,莱布尼茨判别法知收敛,故收敛域为 [1,1)[-1, 1)

内闭一致收敛性有,对 x[1,1)x \in [-1, 1)

S(x)=n=11nxnS(x)=n=1xn1=n=0xn\begin{aligned} S(x) &= \sum_{n=1}^{\infty} \dfrac{1}{n} x^n \\ S'(x) &= \sum_{n=1}^{\infty} x^{n-1} \\ &= \sum_{n=0}^{\infty} x^{n} \end{aligned}

随即对 x(1,1)x \in (-1, 1),有 S(x)=11xS'(x) = \dfrac{1}{1 - x},从而有

S(x)=S(x)S(0)=0xS(t) ⁣dt=ln(1x)+C\begin{aligned} S(x) &= S(x) - S(0)\\ &= \int_0^x S'(t) \d t\\ &= -\ln(1 - x) + C \end{aligned}

代入 x=0x = 0,得 C=0C = 0,从而有 S(x)=ln(1x)S(x) = -\ln(1 - x)

内闭一致收敛性有 S(x)S(x)1-1 处右连续,从而

n=1(1)n11n=S(1)=limx1+S(x)=ln2\begin{aligned} \displaystyle\sum_{n=1}^{\infty} (-1)^{n-1} \dfrac{1}{n} &= - S(-1)\\ &= -\lim\limits_{x \to -1^{+}} S(x) \\ &= \ln 2 \end{aligned}

泰勒级数

既然幂级数可以收敛到一个和函数,能否将一个函数展开成幂级数呢?

假设 f(x)f(x) 在点 x0x_0 的某邻域 Nδ(x0)N_{\delta}(x_0) 内能展成幂级数,即有

f(x)=n=0an(xx0)nxNδ(x0)f(x) = \sum_{n=0}^{\infty} a_n (x - x_0)^n\qquad x \in N_{\delta}(x_0)

显然 f(x)f(x)Nδ(x0)N_{\delta}(x_0) 内有无穷阶导数,且有

f(n)(x0)=n!anf^{(n)}(x_0) = n! a_n

亦即展开式必为

n=0f(n)(x0)n!(xx0)n\sum_{n=0}^{\infty} \dfrac{f^{(n)}(x_0)}{n!} (x - x_0)^n

该幂级数称为 f(x)f(x)x0x_0 处的泰勒级数

f(x)=n=0f(n)(x0)n!(xx0)nf(x) = \sum_{n=0}^{\infty} \dfrac{f^{(n)}(x_0)}{n!} (x - x_0)^n

称为 f(x)f(x)x0x_0 处的泰勒展开式

从而 f(x)f(x)Nδ(x0)N_{\delta}(x_0) 内可展成幂级数的充要条件是 f(x)f(x)Nδ(x0)N_{\delta}(x_0) 内有无穷阶导数,且级数在 Nδ(x0)N_{\delta}(x_0) 内收敛到 f(x)f(x)

f(x)f(x) 在点 x0x_0 的某个邻域 NδN_{\delta} 内有任意阶导数,则 f(x)f(x) 在该邻域内能展成泰勒级数的充要条件为

limnRn(x)=0xNδ(x0)\lim\limits_{n \to \infty}R_n(x) = 0\qquad x \in N_{\delta}(x_0)

这里 Rn(x)R_n(x) 为泰勒级数的余项。

f(x)f(x) 展开为泰勒级数步骤:

  1. 求出 f(n)(x0)f^{(n)}(x_0),若存在 nn 使得 f(n)(x0)f^{(n)}(x_0) 不存在,则 f(x)f(x)x0x_0 处不能展成泰勒级数。
  2. 写出幂级数 n=0f(n)(x0)n!(xx0)n\displaystyle \sum_{n=0}^{\infty} \dfrac{f^{(n)}(x_0)}{n!} (x - x_0)^n,并求出收敛半径 RR
  3. 利用余项 Rn(x)R_n(x) 表达式,考察 xNR(x0)x \in N_{R}(x_0) 时,余项 Rn(x)R_n(x) 极限是否为零。若为零,则展开式为泰勒级数。
    • Rn(x)=f(n+1)(ξ)(n+1)!(xx0)n+1R_n(x) = \dfrac{f^{(n+1)}(\xi)}{(n + 1)!}(x - x_0)^{n+1},其中 ξ\xix0x_0xx 之间。

各种余项:

  • 皮亚诺余项:Rn(x)=o((xx0)n)R_n(x) = o\left((x - x_0)^n\right)
  • 拉格朗日余项:Rn(x)=f(n+1)(ξ)(n+1)!(xx0)n+1R_n(x) = \dfrac{f^{(n+1)}(\xi)}{(n + 1)!}(x - x_0)^{n+1},其中 ξ\xix0x_0xx 之间。
  • 柯西余项:Rn(x)=f(n+1)(θx)n!(1θ)nxn+1R_n(x) = \dfrac{f^{(n+1)}(\theta x)}{n!}(1 - \theta)^nx^{n+1},其中 θ(0,1)\theta \in (0, 1)

二项式级数(mNm \notin \N):

(1+x)m=n=0(mn)xn=1+n=1m(m1)(mn+1)n!xn=1+n=1xnn!i=0n1(mi)\begin{aligned} (1 + x)^m &= \sum_{n=0}^{\infty} \binom{m}{n}x^n\\ &= 1 + \sum_{n=1}^{\infty} \frac{m(m-1)\cdots(m-n+1)}{n!}x^n\\ &= 1 + \sum_{n=1}^{\infty} \dfrac{x^n}{n !} \displaystyle \prod_{i=0}^{n-1}(m - i) \end{aligned}

收敛域需分类讨论:

  1. m>0m > 0[1,1][-1, 1]
  2. 1<m<0-1 < m < 0(1,1](-1, 1]
  3. m1m \le -1(1,1)(-1, 1)

讨论
  1. m>0m > 0

  2. 1<m<0-1 < m < 0

x=1x = -1 时,n>0n > 0 时有

(1)n(mn)=i=0n1m+ii+1=m1m+12m+n1n=mnm+11m+n1n1mn\begin{aligned} (-1)^n \dbinom{m}{n} &= \prod_{i=0}^{n-1}\dfrac{|m| + i}{i + 1}\\ &= \dfrac{|m|}{1} \dfrac{|m| + 1}{2} \cdots \dfrac{|m| + n - 1}{n}\\ &= \dfrac{|m|}{n} \dfrac{|m| + 1}{1} \cdots \dfrac{|m| + n - 1}{n - 1}\\ &\ge \dfrac{|m|}{n} \end{aligned}

(mn)mn\left\lvert \dbinom{m}{n} \right\rvert \ge \dfrac{|m|}{n},故级数发散。

x=1x = 1 时,n=1(mn)=n=11n!i=0n1(mi)\displaystyle \sum_{n=1}^{\infty}\displaystyle \dbinom{m}{n} = \sum_{n=1}^{\infty} \dfrac{1}{n!} \prod_{i=0}^{n-1}(m-i) 为交错级数。而

(mn)=1n!i=0n1(mi)>1n!i=0n1(mi)mnn+1=1(n+1)!i=0n(mi)=(mn+1)\begin{aligned} \dbinom{m}{n} &= \dfrac{1}{n!}\left\lvert \prod_{i=0}^{n-1}(m-i) \right\rvert \\ &> \dfrac{1}{n!}\left\lvert \prod_{i=0}^{n-1}(m-i) \right\rvert \left\lvert \dfrac{m - n}{n + 1} \right\rvert\\ &= \dfrac{1}{(n+1)!} \left\lvert \prod_{i=0}^{n}(m - i) \right\rvert\\ &= \left\lvert \dbinom{m}{n + 1} \right\rvert \end{aligned}

{(mn)}\left\lbrace \left\lvert \dbinom{m}{n} \right\rvert \right\rbrace 是递减数列,莱布尼茨判别法知 n=11n!i=0n1(mi)\displaystyle \sum_{n=1}^{\infty} \dfrac{1}{n !} \displaystyle \prod_{i=0}^{n-1}(m - i),故级数收敛。

  1. m1m \le -1

x=±1x = \pm 1 时,n>0n > 0 时有

(mn)=i=0n1mii+1=i=0n1imi+11\begin{aligned} \left\lvert \dbinom{m}{n} \right\rvert &= \left\lvert \prod_{i=0}^{n-1} \dfrac{m - i}{i + 1} \right\rvert\\ &= \prod_{i=0}^{n-1} \dfrac{i - m}{i + 1}\\ &\ge 1 \end{aligned}

(mn)1\left\lvert \dbinom{m}{n} \ge 1 \right\rvert,故级数发散。

广义积分的敛散性

柯西收敛原理

广义积分 af(x) ⁣dx\displaystyle\int_{a}^{\infty}f(x)\d x 收敛的充要条件是:对任意 ε>0\varepsilon > 0,存在 R>aR > a,使得当 A1,A2>RA_1, A_2 > R 时,恒有

A1A2f(x) ⁣dx<ε\left\lvert \int_{A_1}^{A_2}f(x)\d x \right\rvert < \varepsilon

若广义积分 af(x) ⁣dx\displaystyle\int_{a}^{\infty}|f(x)|\d x 收敛,则称广义积分 af(x) ⁣dx\displaystyle\int_{a}^{\infty}f(x)\d x 绝对收敛;若 af(x) ⁣dx\displaystyle\int_{a}^{\infty}f(x)\d x 收敛而 af(x) ⁣dx\displaystyle\int_{a}^{\infty}|f(x)|\d x 发散,则称 af(x) ⁣dx\displaystyle\int_{a}^{\infty}f(x)\d x 条件收敛

f(x)f(x)[a,)[a, \infty ) 上的非负可积函数,则 af(x) ⁣dx\displaystyle\int_{a}^{\infty}f(x)\d x 收敛的充要条件是:函数 F(x)=axf(t) ⁣dtF(x) = \displaystyle \int_a^x f(t) \d t[a,)[a, \infty) 上有上界。

比较判别法

设函数 f(x),g(x)f(x), g(x) 在区间 [a,)[a, \infty ) 上有定义,且

0f(x)g(x)(xa)0 \le f(x) \le g(x) \quad (x \ge a)

并设 bab \ge a 时,f(x),g(x)f(x), g(x) 在任意区间 [a,b][a, b] 上可积,则有:

  1. ag(x) ⁣dx\displaystyle\int_{a}^{\infty} g(x) \d x 收敛时,af(x) ⁣dx\displaystyle\int_{a}^{\infty} f(x) \d x 收敛;
  2. af(x) ⁣dx\displaystyle\int_{a}^{\infty} f(x) \d x 发散时,ag(x) ⁣dx\displaystyle\int_{a}^{\infty} g(x) \d x 发散。

比较判别法极限形式

设函数 f(x),g(x)f(x), g(x) 在区间 [a,)[a, \infty ) 上有定义,f(x)0,g(x)>0f(x) \ge 0, g(x) > 0

并设 bab \ge a 时,f(x),g(x)f(x), g(x) 在任意区间 [a,b][a, b] 上可积,且

limxf(x)g(x)=λ\lim_{x \to \infty} \dfrac{f(x)}{g(x)} = \lambda

则:

  1. 0λ<0 \le \lambda < \infty ag(x) ⁣dx\displaystyle\int_{a}^{\infty} g(x) \d x 收敛时,af(x) ⁣dx\displaystyle\int_{a}^{\infty} f(x) \d x 收敛;
  2. 0<λ0 < \lambda \le \infty ag(x) ⁣dx\displaystyle\int_{a}^{\infty} g(x) \d x 发散时,af(x) ⁣dx\displaystyle\int_{a}^{\infty} f(x) \d x 发散。

特别地,0<λ<0 < \lambda < \infty 时,两个广义积分同敛散。

柯西判别法

f(x)0f(x) \ge 0x[a,)x \in [a, \infty ),若

limxxpf(x)=λ\lim\limits_{x \to \infty} x^p f(x) = \lambda

则:

  1. 0λ<0 \le \lambda < \infty p>1p > 1 时,af(x) ⁣dx\displaystyle\int_{a}^{\infty} f(x) \d x 收敛;
  2. 0<λ0 < \lambda \le \infty p1p \le 1 时,af(x) ⁣dx\displaystyle\int_{a}^{\infty} f(x) \d x 发散。

狄利克雷判别法

f(x),g(x)f(x), g(x) 在区间 [a,)[a, \infty ) 上有定义,且

  1. 对任意 bab \ge a,积分 abf(x) ⁣dx\displaystyle\int_{a}^{b} f(x) \d x 有界;
  2. 函数 g(x)g(x)[a,)[a, \infty ) 上单调递减且趋于零。

则广义积分 af(x)g(x) ⁣dx\displaystyle\int_{a}^{\infty} f(x)g(x) \d x 收敛。

阿贝尔判别法

f(x),g(x)f(x), g(x) 在区间 [a,)[a, \infty ) 上有定义,且

  1. 广义积分 af(x) ⁣dx\displaystyle\int_{a}^{\infty} f(x)\d x 收敛;
  2. 函数 g(x)g(x)[a,)[a, \infty ) 上单调有界。

则广义积分 af(x)g(x) ⁣dx\displaystyle\int_{a}^{\infty} f(x)g(x) \d x 收敛。

无界积分有类似判别法与定理等,不再赘述。

特殊函数

Γ\Gamma 函数

欧拉积分

含参 ss 的广义积分

0ts1et ⁣dt\int_{0}^{\infty} t^{s-1}\e^{-t} \d t

称为第二类欧拉积分。这也是 Gamma 函数 Γ\Gamma 的定义

Γ(s)=0ts1et ⁣dt\Gamma(s) = \int_{0}^{\infty} t^{s-1}\e^{-t} \d t

Γ\Gamma 函数有性质

Γ(s+1)=sΓ(s)\Gamma(s+1) = s\Gamma(s)


Γ(s+1)=0tset ⁣dt=tset0+s0ts1et ⁣dt=sΓ(s)\begin{aligned} \Gamma(s+1) &= \int_{0}^{\infty} t^{s}\e^{-t} \d t\\ &= -t^s \e^{-t} \as_{0}^{\infty} + s\int_{0}^{\infty} t^{s-1}\e^{-t} \d t\\ &= s\Gamma(s) \end{aligned}

因此 Γ\Gamma 函数可以看作是阶乘的推广,即

Γ(n)=(n1)!\Gamma(n) = (n-1)!

Γ\Gamma 函数及其任意阶导数在 (0,)(0, \infty ) 连续,且

Γ(n)(s)=0ts1et(lnt)n ⁣dt\Gamma^{(n)}(s) = \int_{0}^{\infty} t^{s-1}\e^{-t}(\ln t)^n \d t

余元公式

s(0,1)s \in (0, 1)

Γ(s)Γ(1s)=πsinπs\Gamma(s)\Gamma(1-s) = \dfrac{\pi}{\sin \pi s}

从而得到

Γ(12)=π\Gamma\left(\dfrac{1}{2}\right) = \sqrt{\pi}

B\Beta 函数

含参 x,yx, y 的积分

01tx1(1t)y1 ⁣dt\int_0^1 t^{x-1}(1-t)^{y-1} \d t

  • x1,y1x \ge 1, y \ge 1 时,为定积分
  • x<1x < 1 时,t=0t = 0 为奇点
  • y<1y < 1 时,t=1t = 1 为奇点

并记

B(x,y)=01tx1(1t)y1 ⁣dt\Beta(x, y) = \int_0^1 t^{x-1}(1-t)^{y-1} \d t

为 Beta 函数 B\Beta

B\Beta 函数及其任意阶导数在 (0,)2(0, \infty )^2 连续,且

B(x,y)=Γ(x)Γ(y)Γ(x+y)\Beta(x, y) = \dfrac{\Gamma(x)\Gamma(y)}{\Gamma(x+y)}

B(p,q+1)=qp+qB(p,q)B(p+1,q)=pp+qB(p,q)\Beta(p, q + 1) = \dfrac{q}{p + q}\Beta(p, q)\\ \Beta(p + 1, q) = \dfrac{p}{p + q}\Beta(p, q)

m,nm, n 为自然数时有

B(m,n)=Γ(m)Γ(n)Γ(m+n)=(m1)!(n1)!(m+n1)!\begin{aligned} \Beta(m, n) &= \dfrac{\Gamma(m)\Gamma(n)}{\Gamma(m + n)}\\ &= \dfrac{(m-1)!(n-1)!}{(m+n-1)!} \end{aligned}

计算

I=0π2sinnx ⁣dxI = \int_0^{\frac{\pi}{2}} \sin^n x \d x


过程

{x=n+12y=12\begin{cases} x = \dfrac{n + 1}{2}\\ y = \dfrac{1}{2} \end{cases}

I=12B(n+12,12)=12Γ(n+12)Γ(12)Γ(n+12+12)={(n1)!!n!!π2,n 为偶数(n1)!!n!!,n 为奇数\begin{aligned} I &= \dfrac{1}{2} \Beta\left(\dfrac{n + 1}{2}, \dfrac{1}{2}\right)\\ &= \dfrac{1}{2} \dfrac{\Gamma\left(\dfrac{n + 1}{2}\right)\Gamma\left(\dfrac{1}{2}\right)}{\Gamma\left(\dfrac{n + 1}{2} + \dfrac{1}{2}\right)}\\ &= \begin{cases} \dfrac{(n-1)!!}{n!!} \dfrac{\pi}{2}, & n \text{ 为偶数}\\ \dfrac{(n-1)!!}{n!!}, & n \text{ 为奇数} \end{cases} \end{aligned}

傅里叶级数

三角级数与三角函数系的正交性

1,cosx,sinx,,cosnx,sinnx,1, \cos x, \sin x, \cdots, \cos nx, \sin nx, \cdots

称为基本三角函数系。有如下关系[1]

ππsinnx ⁣dx=0ππcosnx ⁣dx=0ππsinmxcosnx ⁣dx=0ππsinmxsinnx ⁣dx=0(mn)ππcosmxcosnx ⁣dx=0(mn)\begin{aligned} \int_{- \pi}^{\pi} \sin nx \d x &= 0\\ \int_{- \pi}^{\pi} \cos nx \d x &= 0\\ \int_{- \pi}^{\pi} \sin mx \cos nx \d x &= 0\\ \int_{- \pi}^{\pi} \sin mx \sin nx \d x &= 0\quad (m \ne n)\\ \int_{- \pi}^{\pi} \cos mx \cos nx \d x &= 0\quad (m \ne n) \end{aligned}

即三角函数系在区间 [π,π][-\pi, \pi] 上,在「某种含义上」是正交的。

区间 [π,π][-\pi, \pi] 上全体有界可积函数组成的集合 AA 在函数的加法及实数的乘法运算下,构成一个线性空间。

内积

对该线性空间中任意两个函数 f,gf, g,定义内积

(f,g)=1πππf(x)g(x) ⁣dx(f, g) = \dfrac{1}{\pi} \int_{-\pi}^{\pi} f(x) g(x) \d x

并定义

范数

f=(f,f)=1πππf2(x) ⁣dx\|f\| = \sqrt{(f, f)} = \sqrt{\dfrac{1}{\pi} \int_{-\pi}^{\pi} f^{2}(x) \d x}

为函数 ff范数

AA 称为赋范线性空间

两向量 f,gf, g 正交,指它们的内积为 00,函数系正交,即函数系中任意两个不同元素的内积为 00

而注意到

ππ ⁣dx=2πππsin2nx ⁣dx=πππcos2nx ⁣dx=π\begin{aligned} \int_{- \pi}^{\pi} \d x &= 2 \pi\\ \int_{- \pi}^{\pi} \sin^{2} nx \d x &= \pi\\ \int_{- \pi}^{\pi} \cos^{2} nx \d x &= \pi \end{aligned}

因此函数系

12,cosx,sinx,,cosnx,sinnx,\dfrac{1}{\sqrt{2}}, \cos x, \sin x, \cdots, \cos nx, \sin nx, \cdots

不仅正交,而且每个元素范数为 11。因此这个函数系称为单位正交系

展开成傅里叶级数

Aksin(kx+φk)A_{k} \sin(k x + \varphi_{k}) 为项作成的无穷级数

k=0Aksin(kx+φk)\sum_{k=0}^{\infty} A_{k} \sin(k x + \varphi_{k})

称为三角级数。显然级数在长度为 2π2 \pi 的闭区间收敛,和函数 f(x)f(x) 是一个周期函数。

Ansin(nx+φn)A_n \sin(n x + \varphi_n) 可以展开为

Ansin(nx+φn)=Ansinφncosnx+AncosφnsinnxA_n \sin(n x + \varphi_n) = A_n \sin \varphi_n \cos nx + A_n \cos \varphi_n \sin nx

{a02=A0sinφ0an=Ansinφnbn=Ancosφn\begin{cases} \dfrac{a_0}{2} \kern{-0.8em} &= A_0 \sin \varphi_0\\ a_n \kern{-0.8em} &= A_n \sin \varphi_n\\ b_n \kern{-0.8em} &= A_n \cos \varphi_n \end{cases}

nn 阶三角多项式可写为

a02+k=1(akcoskx+bksinkx)\dfrac{a_0}{2} + \sum_{k=1}^{\infty} (a_k \cos kx + b_k \sin kx)

设函数 f(x)f(x) 可展开为三角级数

f(x)=a02+k=1(akcoskx+bksinkx)f(x) = \dfrac{a_0}{2} + \sum_{k=1}^{\infty} (a_k \cos kx + b_k \sin kx)

则期望求得系数 a0,a1,b1,a_0, a_1, b_1, \cdots 与函数 f(x)f(x) 的关系。设级数一致收敛于 f(x)f(x),则

ππf(x) ⁣dx=a022π=a0π\int_{- \pi}^{\pi} f(x) \d x = \dfrac{a_0}{2} \cdot 2 \pi = a_0 \pi

a0=1πππf(x) ⁣dxa_0 = \dfrac{1}{\pi} \int_{- \pi}^{\pi} f(x) \d x

类似线代中的「选择」方法:对某个向量 v\bm{v},为选取其第 ii 个分量,将其与对应的基向量 ei\bm{e}_i 进行内积 vei\bm{v} \boldsymbol{\cdot} \bm{e}_i,从而得到系数。

因此套用这个方法,有

an=(f(x),cosnx)=1πππf(x)cosnx ⁣dx\begin{aligned} a_n &= \left(f(x), \cos nx\right)\\ &= \dfrac{1}{\pi} \int_{- \pi}^{\pi} f(x) \cos nx \d x \end{aligned}

bn=(f(x),sinnx)=1πππf(x)sinnx ⁣dx\begin{aligned} b_n &= \left(f(x), \sin nx\right)\\ &= \dfrac{1}{\pi} \int_{- \pi}^{\pi} f(x) \sin nx \d x \end{aligned}

a02=A0sinφ0\dfrac{a_0}{2} = A_0 \sin \varphi_0 也有简化系数表示的意义所在。

从而有

欧拉-傅里叶公式

{an=1πππf(x)cosnx ⁣dx,(n=0,1,2,)bn=1πππf(x)sinnx ⁣dx,(n=1,2,)\begin{cases} a_n \kern{-0.8em} &= \dfrac{1}{\pi} \displaystyle \int_{-\pi}^{\pi} f(x) \cos nx \d x,\quad (n = 0, 1, 2, \cdots)\\ b_n \kern{-0.8em} &= \dfrac{1}{\pi} \displaystyle \int_{-\pi}^{\pi} f(x) \sin nx \d x,\quad (n = 1, 2, \cdots) \end{cases}

a0,a1,b1,a_0, a_1, b_1, \cdots 称为函数 f(x)f(x)傅里叶系数(Fourier),三角级数

a02+k=1(akcoskx+bksinkx)\dfrac{a_0}{2} + \sum_{k=1}^{\infty} (a_k \cos kx + b_k \sin kx)

称为 f(x)f(x)傅里叶级数,记为

f(x)a02+k=1(akcoskx+bksinkx)f(x) \sim \dfrac{a_0}{2} + \sum_{k=1}^{\infty} (a_k \cos kx + b_k \sin kx)

不用 ==,而用 \sim 的原因是因为傅里叶级数不一定收敛于原函数。

狄利克雷收敛定理

f(x)f(x) 是周期为 2π2 \pi 的周期函数,若

  1. f(x)f(x)[π,π][-\pi, \pi] 上连续或只有有限个第一类间断点
  2. f(x)f(x)[π,π][-\pi, \pi] 上至多只有有限个严格极值点,即 f(x)f(x)[π,π][-\pi, \pi] 上分段单调

f(x)f(x) 的傅里叶级数收敛,且傅里叶级数 S(x)S(x)

S(x)={f(x),x 为 f(x) 的连续点f(x+)+f(x)2,x 为 f(x) 的间断点S(x) = \begin{cases} f(x), & x \text{ 为 } f(x) \text{ 的连续点}\\ \dfrac{f(x^{+})+f(x^{-})}{2}, & x \text{ 为 } f(x) \text{ 的间断点} \end{cases}

f(x)f(x) 是偶函数,则

bn=1πππf(x)sinnx ⁣dx=0\begin{aligned} b_n &= \dfrac{1}{\pi} \int_{- \pi}^{\pi} f(x) \sin nx \d x\\ &= 0 \end{aligned}

此时 f(x)f(x) 的傅里叶级数仅含余弦函数,即

f(x)a02+k=1akcoskxf(x) \sim \dfrac{a_0}{2} + \sum_{k=1}^{\infty} a_k \cos kx

称为余弦级数

f(x)f(x) 是奇函数,则

an=1πππf(x)cosnx ⁣dx=0\begin{aligned} a_n &= \dfrac{1}{\pi} \int_{- \pi}^{\pi} f(x) \cos nx \d x\\ &= 0 \end{aligned}

此时 f(x)f(x) 的傅里叶级数仅含正弦函数,即

f(x)k=1bksinkxf(x) \sim \sum_{k=1}^{\infty} b_k \sin kx

称为正弦级数

设定义域为 R\R 的周期函数 f(x)f(x)x[π,π]x \in [-\pi, \pi] 时有 f(x)=x2f(x) = x^2,求 n=11n2\displaystyle \sum_{n=1}^{\infty}\dfrac{1}{n^2}n=1(1)n11n2\displaystyle \sum_{n=1}^{\infty}(-1)^{n-1} \dfrac{1}{n^2}


过程

由于 f(x)f(x) 是偶函数,因此只含余弦级数。由傅里叶级数的定义,有

a0=1πππf(x) ⁣dx=2π0πx2 ⁣dx=23π2\begin{aligned} a_0 &= \dfrac{1}{\pi} \int_{-\pi}^{\pi} f(x) \d x\\ &= \dfrac{2}{\pi} \int_{0}^{\pi} x^2 \d x\\ &= \dfrac{2}{3} \pi^2 \end{aligned}

an=1πππf(x)cosnx ⁣dx=2π0πx2cosnx ⁣dx=2π0πx2n ⁣dsinnx=2nπ(x2sinnx0π0π2xsinnx ⁣dx)=4n2π0πx ⁣dcosnx=4n2π(xcosnx0π0πcosnx ⁣dx)=4n2π(1)nπ=4n2(1)n\begin{aligned} a_n &= \dfrac{1}{\pi} \int_{-\pi}^{\pi} f(x) \cos nx \d x\\ &= \dfrac{2}{\pi} \int_{0}^{\pi} x^2 \cos nx \d x\\ &= \dfrac{2}{\pi} \int_0^{\pi} \dfrac{x^2}{n} \d \sin nx\\ &= \dfrac{2}{n\pi} \left(x^2 \sin nx \as_0^{\pi} - \int_0^{\pi} 2x \sin nx \d x\right)\\ &= \dfrac{4}{n^2 \pi} \int_0^{\pi} x \d \cos nx\\ &= \dfrac{4}{n^2 \pi} \left(x \cos nx \as_0^{\pi} - \int_0^{\pi} \cos nx \d x\right)\\ &= \dfrac{4}{n^2 \pi} \cdot (-1)^n \pi\\ &= \dfrac{4}{n^2}(-1)^n \end{aligned}

狄利克雷收敛定理有

x2=π23+n=14n2(1)ncosnxx^2 = \dfrac{\pi^2}{3} + \sum_{n=1}^{\infty} \dfrac{4}{n^2}(-1)^n \cos nx

因此取 x=πx = \pi

π2=π23+n=14n2\pi^2 = \dfrac{\pi^2}{3} + \sum_{n=1}^{\infty} \dfrac{4}{n^2}

n=11n2=π26\sum_{n=1}^{\infty} \dfrac{1}{n^2} = \dfrac{\pi^2}{6}

这也是巴塞尔问题的解。

x=0x = 0

0=π23+n=1(1)n4n20 = \dfrac{\pi^2}{3} + \sum_{n=1}^{\infty} (-1)^n \dfrac{4}{n^2}

n=1(1)n11n2=π212\sum_{n=1}^{\infty} (-1)^{n-1}\dfrac{1}{n^2} = \dfrac{\pi^2}{12}

任意周期的周期函数的傅里叶级数

设周期为 2l2l 的周期函数 f(x)f(x) 满足狄利克雷收敛定理的条件,则 f(x)f(x) 的傅里叶级数

a02+n=1(ancosnπxl+bnsinnπxl)\dfrac{a_0}{2} + \sum_{n=1}^{\infty} \left(a_n \cos \dfrac{n \pi x}{l} + b_n \sin \dfrac{n \pi x}{l}\right)

收敛,且

S(x)={f(x),x 为 f(x) 的连续点f(x+)+f(x)2,x 为 f(x) 的间断点S(x) = \begin{cases} f(x), & x \text{ 为 } f(x) \text{ 的连续点}\\ \dfrac{f(x^{+})+f(x^{-})}{2}, & x \text{ 为 } f(x) \text{ 的间断点} \end{cases}

其中

{an=1lllf(x)cosnπxl ⁣dx,(n=0,1,2,)bn=1lllf(x)sinnπxl ⁣dx,(n=1,2,)\begin{cases} a_n \kern{-0.8em} &= \dfrac{1}{l} \displaystyle \int_{-l}^{l} f(x) \cos \dfrac{n \pi x}{l} \d x,\quad (n = 0, 1, 2, \cdots)\\ b_n \kern{-0.8em} &= \dfrac{1}{l} \displaystyle \int_{-l}^{l} f(x) \sin \dfrac{n \pi x}{l} \d x,\quad (n = 1, 2, \cdots) \end{cases}


  1. 证明见线代笔记。难怪线代课会讲积分,太愚钝了当时没能注意到这一层联系。 ↩︎